Tải bản đầy đủ (.pdf) (50 trang)

(Luận văn thạc sĩ) Ứng dụng của luật thuận nghịch thặng dư bậc hai

Bạn đang xem bản rút gọn của tài liệu. Xem và tải ngay bản đầy đủ của tài liệu tại đây (309.69 KB, 50 trang )

ĐẠI HỌC THÁI NGUYÊN
TRƯỜNG ĐẠI HỌC KHOA HỌC
——————–o0o——————–

ĐỖ THỊ THOA

ỨNG DỤNG CỦA LUẬT THUẬN NGHỊCH VÀ
THẶNG DƯ BẬC HAI

THÁI NGUYÊN - 2019


ĐẠI HỌC THÁI NGUYÊN
TRƯỜNG ĐẠI HỌC KHOA HỌC
——————–o0o——————–

ĐỖ THỊ THOA

ỨNG DỤNG CỦA LUẬT THUẬN NGHỊCH VÀ
THẶNG DƯ BẬC HAI

CHUYÊN NGÀNH: PHƯƠNG PHÁP TOÁN SƠ CẤP
MÃ SỐ: 8 46 01 13

LUẬN VĂN THẠC SĨ TOÁN HỌC

NGƯỜI HƯỚNG DẪN KHOA HỌC

PGS. TS. NGUYỄN VĂN HOÀNG

THÁI NGUYÊN - 2019




i

Mục lục
Mở đầu

1

1 Kiến thức chuẩn bị

3

1.1

Định lý Fermat nhỏ và định lý Euler . . . . . . . . . . . . . . . . .

3

1.2

Sơ lược về phương trình đồng dư . . . . . . . . . . . . . . . . . . .

5

2 Ứng dụng của luật thuận nghịch và thặng dư bậc hai
2.1

2.2


10

Thặng dư bậc hai và ứng dụng . . . . . . . . . . . . . . . . . . . . 10
2.1.1

Phương trình đồng dư bậc hai . . . . . . . . . . . . . . . . 10

2.1.2

Thặng dư bậc hai . . . . . . . . . . . . . . . . . . . . . . . . 12

2.1.3

Tiêu chuẩn Euler và ký hiệu Legendre . . . . . . . . . . . . 16

2.1.4

Bổ đề Gauss . . . . . . . . . . . . . . . . . . . . . . . . . . . 21

2.1.5

Một số ứng dụng khác . . . . . . . . . . . . . . . . . . . . . 27

Luật thuận nghịch bậc hai và ứng dụng . . . . . . . . . . . . . . . 32
2.2.1

Luật thuận nghịch bậc hai . . . . . . . . . . . . . . . . . . . 32

2.2.2


Ứng dụng của luật thuận nghịch bậc hai . . . . . . . . . . 37

Kết luận

46

Tài liệu tham khảo

47


1

Mở đầu
Có thể nói Số học là lĩnh vực xuất hiện sớm nhất trong lịch sử Toán học,
nó ra đời từ khi con người bắt đầu làm việc với những con số. Số học là một
phân môn quan trọng trong toán học đã gắn bó với tất cả chúng ta xuyên suốt
quá trình học toán từ bậc Tiểu học đến Trung học phổ thông. Sự kì diệu của
Số học thường tiềm ẩn những thử thách sâu sắc để thách thức trí tuệ của con
người. Trong các thành tựu của số học thì luật thuận nghịch và thặng dư bậc
hai là một nội dung quan trọng. Đây là những mảng kiến thức liên quan đến lý
thuyết đồng dư và có nhiều ứng dụng trong việc giải các bài toán số học hay và
khó liên quan đến tính giải được của phương trình đồng dư bậc hai. Nội dung
này cho phép ta xác định tính giải được của phương trình đồng dư bậc hai bất
kỳ, tuy nhiên nó không cung cấp một phương pháp hiệu quả để tìm nghiệm.
Luật thuận nghịch bậc hai (hay còn gọi là luật thuận nghịch của các thặng dư
bậc hai) được tiên đoán bởi Euler và Legendre và lần đầu tiên được chứng minh
thuyết phục bởi Gauss. Gauss gọi đó là "định lý vàng" và rất tự hào về nó đến
mức ông tiếp tục tìm ra tám chứng minh khác cho nó cho đến cuối đời.
Đề tài luận văn "Ứng dụng của luật thuận nghịch và thặng dư bậc hai" có

mục đích là hệ thống lại mảng kiến thức liên quan đến luật thuận nghịch và
thặng dư bậc hai, từ đó trình bày một số ví dụ ứng dụng hay của chúng nhằm
cung cấp một tài liệu tốt để dạy và học cho giáo viên và học sinh phổ thông
trung học.
Luận văn ngoài phần mở đầu, kết luận thì nội dung chính gồm 2 chương
trình bày lại một cách hệ thống về luật thuận nghịch và thặng dư bậc hai cùng
một số ứng dụng của chúng, với bố cục cụ thể như sau:
Chương 1. Kiến thức chuẩn bị. trình bày phát biểu và chứng minh định


2

lý Fermat nhỏ, định lý Euler. Trình bày khái niệm và cách giải phương trình
đồng dư tuyến tính, hệ phương trình đồng dư tuyến tính (định lý thặng dư
Trung Hoa).
Chương 2. Ứng dụng của luật thuận nghịch và thặng dư bậc hai.
Chương 2 trình bày định nghĩa và các tính chất của phương trình đồng dư bậc
hai, thặng dư bậc hai, cách tính bằng định nghĩa, cách tính thông qua ký hiệu
Legendre, cách tính thông qua luật thuận nghịch bậc hai. Sau đó ứng dụng
thặng dư bậc hai và luật thuận nghịch bậc hai để tính toán và giải một số bài
toán chứng minh, tìm căn nguyên thủy, kiểm tra tính nguyên tố.
Luận văn được hoàn thành tại trường Đại học Khoa học, Đại học Thái
Nguyên. Lời đầu tiên tác giả xin được bày tỏ lòng biết ơn sâu sắc tới thầy giáo
PGS.TS. Nguyễn Văn Hoàng. Thầy đã dành nhiều thời gian hướng dẫn cũng
như giải đáp các thắc mắc của tôi trong suốt quá trình làm luận văn. Tôi xin
bày tỏ lòng biết ơn sâu sắc tới thầy.
Tác giả xin chân thành cảm ơn toàn thể các thầy cô trong Khoa Toán - Tin,
trường Đại học Khoa học - Đại học Thái Nguyên đã tận tình hướng dẫn, truyền
đạt kiến thức trong suốt thời gian theo học, thực hiện và hoàn thành luận văn.
Cảm ơn sự giúp đỡ của bạn bè, người thân và các đồng nghiệp trong thời

gian làm luận văn.
Thái Nguyên, tháng 05 năm 2019
Người viết luận văn

Đỗ Thị Thoa


3

Chương 1
Kiến thức chuẩn bị
Chương 1 trình bày phát biểu và chứng minh định lý Fermat nhỏ, định lý
Euler. Trình bày khái niệm và cách giải phương trình đồng dư tuyến tính, hệ
phương trình đồng dư tuyến tính (định lý thặng dư Trung Hoa). Các kiến thức
ở chương này giúp việc trình bày ở chương sau được hệ thống và dễ theo dõi
hơn.

1.1

Định lý Fermat nhỏ và định lý Euler

Mục này trình bày hai định lý quan trọng trong lý thuyết đồng dư là định lý
Fermat nhỏ và định lý Euler.
Định lý 1.1.1 (Định lý Fermat nhỏ). Cho p là số nguyên tố và a là số nguyên.
Nếu p a thì
ap−1 ≡ 1

(mod p).

Chứng minh. Xét p − 1 số nguyên a, 2a, 3a, . . . , (p − 1)a. Chú ý rằng p


ia với

mọi i = 1, 2, . . . , p − 1 (vì nếu ngược lại, tồn tại 1 ≤ i ≤ p − 1 để p | ia, kéo theo
p | a hoặc p | i; nhưng vì p

a, nên ta có p | i điều này mâu thuẫn). Cũng chú

ý rằng không có hai số nào trong p − 1 số nguyên a, 2a, 3a, . . . , (p − 1)a đồng dư
modulo p (vì nếu ngược lại, thì tồn tại nghịch đảo a của a modulo p; nên từ
ia ≡ ja (mod p) với i = j thì iaa ≡ jaa (mod p), từ đó i ≡ j (mod p), điều này

là không thể). Do đó tập các số dư trong phép chia cho p của các số nguyên
a, 2a, 3a, . . . , (p − 1)a phải là {1, 2, 3, . . . , p − 1}. Khi đó,
(a)(2a)(3a) · · · (p − 1)a ≡ (1)(2)(3) · · · (p − 1)

(mod p)


4

hay tương đương với
ap−1 (p − 1)! ≡ (p − 1)!

(mod p).

Mặt khác ta thấy (p − 1)! và p là hai số nguyên tố cùng nhau, nên từ đồng dư
bên trên cho ta thấy ap−1 ≡ 1 (mod p), điều phải chứng minh.
Ví dụ 1.1.2. Với a = 3, p = 5 suy ra 34 = 81 ≡ 1 (mod 5). Tương tự, ta tính
được 910 ≡ 1 (mod 11).

Tiếp theo ta trình bày định lý Euler đó là một dạng tổng quát hoá của định
lý Fermat nhỏ.
Định lý 1.1.3 (Định lý Euler). Nếu m là số nguyên dương và a là số nguyên
sao cho a nguyên tố cùng nhau với m, thì
aφ(m) ≡ 1

(mod m),

trong đó φ(m) là ký hiệu của phi hàm Euler (hàm này đếm số các số nguyên
trong phạm vi từ 1 đến m mà nguyên tố cùng nhau với m).
Chứng minh. Gọi r1 , r2 , . . . , rφ(m) là φ(m) số nguyên dương không lớn hơn m
sao cho (ri , m) = 1 với i = 1, 2, . . . , φ(m). Xét φ(m) số nguyên xác định bởi
r1 a, r2 a, . . . , rφ(m) a. Chú ý rằng (ri a, m) = 1 với mọi i = 1, 2, . . . , φ(m) (vì nếu

ngược lại, tồn tại i để (ri a, m) > 1 thì tồn tại ước nguyên tố p của (ri a, m), từ đó
p | ri a và p | m. Ta có p | ri a kéo theo p | ri hoặc p | a; nên hoặc ta có p | ri và
p | m hoặc ta có p | a và p | m. Nhưng p | ri và p | m là không thể vì (ri , m) = 1;

còn lại nếu p | a và p | m thì đó cũng là không thể vì (a, m) = 1). Ngoài ra cũng
chú ý rằng không có hai số nào trong các số r1 a, r2 a, . . . , rφ(m) a đồng dư modulo
m (vì nếu ngược lại thì do (a, m) = 1, nên tồn tại nghịch đảo modulo a của a.

Do đó, nếu ri a ≡ rj a (mod m) với i = j, kéo theo ri aa ≡ rj aa (mod m), từ đó
ri ≡ rj (mod m), điều này là không thể). Do đó tập các số dư khi chia cho m của

các số nguyên r1 a, r2 a, . . . , rφ(m) a là {r1 , r2 , . . . , rφ(m) }. Do đó, ta có
(r1 a)(r2 a) · · · (rφ(m) a) ≡ r1 r2 · · · rφ(m)

(mod m),



5

hay tương đương với
aφ(m) r1 r2 · · · rφ(m) ≡ r1 r2 · · · rφ(m)

(mod m).

Bây giờ, m | (aφ(m) r1 r2 · · · rφ(m) − r1 r2 · · · rφ(m) ) kéo theo m | (r1 r2 · · · rφ(m) ) ×
(aφ(m) − 1). Chú ý vì (ri , m) = 1 với i = 1, 2, . . . , φ(m) nên (r1 r2 . . . rφ(m) , m) = 1.

Do đó ta suy ra m | (aφ(m) − 1) và do đó aφ(m) ≡ 1 (mod m), điều phải chứng
minh.
Định lý Euler là tổng quát hóa của định lý nhỏ Fermat vì nếu n = p là số
nguyên tố thì φ(p) = p − 1. Định lý này có thể được sử dụng để dễ dàng giản ước
với những modulo m rất lớn.
Ví dụ 1.1.4. Ví dụ tìm chữ số tận cùng của số 7222 .
Giải. Chú ý rằng 7 và 10 là nguyên tố cùng nhau và φ(10) = 4. Bởi vậy 74 ≡ 1
(mod 10). Và ta có
7222 ≡ 74.55+2 72 ≡ 155 72 ≡ 49 ≡ 9

(mod 10).

Vậy 7222 có chữ số tận cùng là 9.

1.2

Sơ lược về phương trình đồng dư

Định nghĩa 1.2.1. Phương trình đồng dư đại số bậc n là một đồng dư thức có

dạng
f (x) = a0 xn + a1 xn−1 + . . . + an ≡ 0

(mod m)

(1.1)

trong đó x là ẩn, ai ∈ Z (với i = 1, 2, . . . , n) và a0 ≡ 0 (mod m).
Chú ý 1.2.2. (i) Giải phương trình (1.1) là tìm tất cả các giá trị nguyên của
x thoả mãn đồng dư thức (1.1). Nếu x = x0 thoả mãn phương trình (1.1) thì

mọi số x ≡ x0 (mod m) đều thoả mãn (1.1); trong trường hợp này tập hợp
{x ∈ Z | x ≡ x0 (mod m)} được gọi là một nghiệm của phương trình đồng dư

(1.1), kí hiệu là x0 hoặc x ≡ x0 (mod m).


6

(ii) Số nghiệm của phương trình (1.1) là số các phần tử trong một hệ thặng dư
đầy đủ theo modulo m mà thỏa mãn (1.1).
(iii) Hai phương trình đồng dư được gọi là tương đương nếu tập hợp các số
nguyên thỏa mãn các phương trình đó là trùng nhau.
Ví dụ 1.2.3. Xét phương trình x2 ≡ 1 (mod 5).
Giải. Ta thấy trong các số 0, 1, 2, 3, 4 của hệ thặng dư không âm bé nhất theo
modulo 5, có hai số 1 và 4 thỏa mãn phương trình đã cho. Vậy phương trình có
hai nghiệm là x ≡ 1 (mod 5) và x ≡ 4 (mod 5).
Ví dụ 1.2.4. Giải phương trình đồng dư x4 + 7x + 4 ≡ 0 (mod 9).
Giải. Dễ thấy phương trình x4 + 7x + 4 ≡ 0 (mod 3) có nghiệm là x ≡ 1 (mod 3)
(hay x = 3t + 1 với t ∈ Z). Thay x vào phương trình cần giải và bỏ đi những số

hạng chia hết cho 9 ta được
6t + 3 ≡ 0

(mod 9)

⇔ 2t + 1 ≡ 0

(mod 3)

⇔t≡1

(mod 3)

⇔ t = 3k + 1.

Vậy phương trình có nghiệm là x = 3(3k + 1) + 1 hay x ≡ 4 (mod 9).
Định nghĩa 1.2.5. Phương trình đồng dư ax ≡ b (mod m) được gọi là phương
trình đồng dư tuyến tính với a, b, m là các số nguyên đã biết. Khi đó x ≡ x0
(mod m) là một nghiệm của phương trình khi và chỉ khi ax0 ≡ b (mod m).

Định lý 1.2.6 ([6]). Phương trình đồng dư tuyến tính ax ≡ b (mod m) có nghiệm
khi và chỉ khi d | b, trong đó d = (a, m). Nếu d | b thì phương trình có d nghiệm.
Hệ quả 1.2.7 ([6]). Phương trình đồng dư tuyến tính ax ≡ b (mod m) có nghiệm
duy nhất khi và chỉ khi (a, m) = 1.
Hệ quả 1.2.8 ([6]). Cho phương trình đồng dư tuyến tính ax ≡ b (mod m) và
gọi d = (a, m). Nếu d | b thì d nghiệm modulo m của phương trình là
(x0 +

mt
)

d

(mod m) với t = 0, 1, 2, . . . , d − 1


7

trong đó x0 là một số nguyên thỏa mãn của phương trình ad x ≡

b
d

(mod

m
d ).

Ví dụ 1.2.9. Giải phương trình đồng dư 12x ≡ 7 (mod 23).
Giải. Ta có (12, 23) =1 nên phương trình có nghiệm duy nhất. Phương trình đã
cho tương đương với 12x = 23t + 7 (t ∈ Z). Lấy t = 7 suy ra x = 14. Vậy nghiệm
của phương trình đã cho là x ≡ 14 (mod 23).
Ví dụ 1.2.10. Giải phương trình đồng dư 17x ≡ 13 (mod 11).
Giải. Ta có 17 ≡ 6 (mod 11) suy ra
17x ≡ 6x (mod 11).

(1.2)

Mặt khác
13 ≡ 2


(mod 11).

(1.3)

Từ (1.2) và (1.3) và theo tính chất bắc cầu ta có 6x ≡ 2 (mod 11). Do (2, 11) =1
nên giản ước hai vế cho 2 ta được 3x ≡ 1 (mod 11) hay 3x = 11t + 1. Lấy t = 1
suy ra x = 4. Do (3, 11) = 1 nên phương trình có nghiệm duy nhất là x ≡ 4
(mod 11).

Định lý 1.2.11 (Định lý thặng dư Trung Hoa, [5]). Cho m1 , m2 , . . . , mn là các số
nguyên dương đôi một nguyên tố cùng nhau và cho b1 , b2 , . . . , bn là các số nguyên
bất kỳ. Khi đó hệ phương trình đồng dư tuyến tính

x ≡ b1 (mod m1 )



x ≡ b2 (mod m2 )
·········



x ≡ bn (mod mn )

có nghiệm duy nhất modulo m1 m2 · · · mn .
Chứng minh. Đầu tiên ta xây dựng một số nguyên thỏa mãn hệ đã cho. Đặt
M = m1 m2 · · · mn . Với mỗi i = 1, 2, . . . , n đặt Mi = M/mi . Bây giờ với mỗi
i = 1, 2, . . . , n ta có (Mi , mi ) = 1. Do đó theo Hệ quả 1.2.7, phương trình Mi x ≡ 1
(mod mi ) có một nghiệm xi (mod mi ). Đặt
x = b1 M1 x1 + b2 M2 x2 + · · · + bn Mn xn .



8

Chú ý rằng x là một nghiệm của hệ đã cho vì với mỗi i = 1, 2, . . . , n, ta có
x = b1 M1 x1 + b2 M2 x2 + · · · + bn Mn xn
≡ 0 + 0 + · · · + bi + · · · + 0
≡ bi

(mod mi )

(mod mi ).

Ta còn phải chứng minh tính duy nhất của nghiệm modulo M . Gọi x là một
nghiệm khác của hệ. Khi đó, với mọi i, ta có x = bi (mod mi ); vì x ≡ bi (mod mi )
với mọi i, ta có x ≡ x (mod mi ) với mọi i, hay tương đương, mi | (x − x ) với mọi
i. Khi đó, M | (x − x ), từ đó x ≡ x (mod M ). Định lý được chứng minh.

Ví dụ 1.2.12. Giải hệ phương trình đồng dư

x ≡ 2 (mod 3)
x≡1
x≡3



(mod 4)
(mod 5).

Giải. Sử dụng các ký hiệu trong chứng minh của Định lý 1.2.11, ta có

M = 3 · 4 · 5 = 60
M
60
M1 =
=
= 20
m1
3
M
60
M2 =
=
= 15
m2
4
M
60
M1 =
=
= 12
m3
5

Ta giải các phương trình đồng dư Mi xi ≡ 1 (mod mi ), i = 1, 2, 3. M1 x1 ≡ 1
(mod m1 ) trở thành 20x1 ≡ 1 (mod 3), hay 2x1 ≡ 1 (mod 3), từ đó x1 ≡ 2 (mod 3).
M2 x2 ≡ 1 (mod m2 ) trở thành 15x2 ≡ 1 (mod 4), hay 3x2 ≡ 1 (mod 4), từ đó
x2 ≡ 3 (mod 4). Cuối cùng, M3 x3 ≡ 1 (mod m3 ) trở thành 12x3 ≡ 1 (mod 5), hay
2x3 ≡ 1 (mod 5), từ đó x3 ≡ 3 (mod 5). Do đó, ta đặt
x1 = 2,


x2 = 3,

x3 = 3.

Cuối cùng, ta tính nghiệm x của hệ
x = b1 M1 x1 + b2 M2 x2 + b3 M3 x3


9

= 2 · 20 · 2 + 1 · 15 · 3 + 3 · 12 · 3
= 233
≡ 53

(mod 60).

Vậy 53 (mod 60) là nghiệm của hệ đã cho.


10

Chương 2
Ứng dụng của luật thuận nghịch và
thặng dư bậc hai
Chương này trình bày định nghĩa và các tính chất của thặng dư bậc hai: cách
tính thặng dư bằng định nghĩa, cách tính thặng dư thông qua ký hiệu Legendre,
cách tính thông qua luật thuận nghịch bậc hai. Sau đó ứng dụng thặng dư bậc
hai và luật thuận nghịch bậc hai để tính toán và giải một số bài toán chứng
minh, tìm căn nguyên thủy, kiểm tra tính nguyên tố.


2.1

Thặng dư bậc hai và ứng dụng

Thặng dư bậc hai đóng vai trò rất quan trọng trong lý thuyết số. Chẳng hạn,
thuật toán phân tích số nguyên ra thừa số nguyên tố. Ngoài ra thặng dư bậc
hai cũng ứng dụng lớn trong mật mã cũng như trong các giao thức mã hóa....
Ở đây ta xét ứng dụng liên quan đến giải phương trình đồng dư bậc hai trong
lý thuyết và thực hành.

2.1.1

Phương trình đồng dư bậc hai

Xét phương trình đồng dư bậc hai theo modulo nguyên tố (theo tài liệu [5]).
Ax2 + Bx + C ≡ 0

(mod p),

(2.1)

trong đó p nguyên tố lẻ và A, B, C ∈ Z với p A. (Nếu p | A thì quay về phương
trình tuyến tính). Vì p nguyên tố lẻ và p A, nên p 4A. Ta nhân hai vế của


11

phương trình với 4A ta được
4A(Ax2 + Bx + C) ≡ 0


(mod p).

(2.2)

Nhưng ta có
4A(Ax2 + Bx + C) = 4A2 x2 + 4ABx + 4AC
= (2Ax + B)2 − (B 2 − 4AC).

Do đó đồng dư thức (2.2) được viết lại thành
(2Ax + B)2 ≡ (B 2 − 4AC)

(mod p)

nó có dạng
y 2 ≡ a (mod p)

(2.3)

trong đó y = 2Ax + B và a = B 2 − 4AC .
Nếu phương trình y 2 ≡ a (mod p) có nghiệm, thì ta suy ra phương trình
2Ax + B = y có nghiệm x modulo p (vì (2A, p) = 1). Do đó phương trình (2.1) có

nghiệm nếu và chỉ nếu phương trình (2.3) có nghiệm.
Ví dụ sau sẽ minh họa điều này.
Ví dụ 2.1.1. Giải phương trình đồng dư bậc hai 3x2 − 4x + 7 ≡ 0 (mod 13).
Giải. Nhân hai vế của phương trình với 4.3 = 12 ta được
36x2 − 48x + 84 ≡ 0

(mod 13).


(6x − 4)2 ≡ (16 − 84)

(mod 13)

Tức là

(6x − 4)2 ≡ 10

(mod 13)

Đặt y = 6x − 4. Khi đó y 2 ≡ 10 (mod 13). Phương trình đồng dư này có chính xác
hai nghiệm y ≡ 6 (mod 13) và y ≡ 7 (mod 13). Do đó, các nghiệm của phương
trình được cho bởi các phương trình bậc nhất. Ta có 6x − 4 ≡ 6 (mod 13) và 6x −
4 ≡ 7 (mod 13), chúng có hai nghiệm là x ≡ 6 (mod 13) và x ≡ 4 (mod 13).


12

Chú ý rằng ví dụ trên có chính xác hai nghiệm. Nhưng ví dụ tiếp theo chỉ ra
rằng không phải mọi phương trình đồng dư bậc hai đều có nghiệm.
Ví dụ 2.1.2. Giải phương trình đồng dư bậc hai 3x2 + 7x + 5 ≡ 0 (mod 13).
Giải. Ta có
3x2 + 7x + 5 ≡ 0

(mod 13)

⇔ 36x2 + 84x + 60 ≡ 0
⇔ (6x + 7)2 ≡ (49 − 60)
⇔ (6x + 7)2 ≡ −11
⇔ (6x + 7)2 ≡ 2


(mod 13)
(mod 13)

(mod 13)

(mod 13).

Nhưng không có bình phương của thặng dư dương nhỏ nhất nào theo modulo
13 là bằng 2. Do đó phương trình đã cho không có nghiệm.

2.1.2

Thặng dư bậc hai

Từ kết quả trình bày ở Mục 2.1.1, ta thấy rằng có các câu hỏi mới nảy sinh
đó là: “Khi nào thì phương trình đồng dư x2 ≡ a (mod p) (đã đề cập ở (2.3)) có
nghiệm? Khi phương trình đó có nghiệm, thì số nghiệm của nó là bao nhiêu?”
Để dần dần trả lời cho câu hỏi trên ta sẽ tìm hiểu khái niệm thặng dư bậc
hai và nghiên cứu tính chất của nó để áp dụng vào giải phương trình đồng dư
bậc hai x2 ≡ a (mod p). Trước tiên là định nghĩa thặng dư bậc hai như sau.
Định nghĩa 2.1.3 ([5]). Cho m là một số nguyên và a là một số nguyên sao
cho (m, a) = 1. Khi đó số a được gọi là một thặng dư bậc hai của m nếu tồn tại
một số nguyên x sao cho x2 ≡ a (mod m); trường hợp ngược lại ta nói a thặng
dư không bậc hai của m.
Chú ý rằng nếu b ≡ a (mod m), và nếu a là một thặng dư bậc hai của m, thì
b cũng là một thặng dư bậc hai của m. Do đó, không mất tính tổng quát, ta chỉ

cần tập trung vào xét các thặng dư dương nhỏ nhất modulo m.
Ví dụ sau đây sẽ minh họa cho định nghĩa trên.

Ví dụ 2.1.4. Tìm các thặng dư bậc hai và thặng dư không bậc hai của p = 6.


13

Giải. Ta có
12 ≡ 1

(mod 6)

22 ≡ 4

(mod 6)

33 ≡ 3

(mod 6)

42 ≡ 4 ≡ 22

(mod 6)

52 ≡ 1

(mod 6).

Theo đó, 6 có chính xác 3 thặng dư bậc hai là 1, 3, 4. Và nó có 2 thặng dư không
bậc hai là 2, 5.
Ví dụ 2.1.5. Tìm các thặng dư bậc hai và thặng dư không bậc hai của p = 13.
Giải. Ta nhận thấy rằng

12 ≡ 1 ≡ 122

(mod 13)

22 ≡ 4 ≡ 112

(mod 13)

32 ≡ 9 ≡ 102

(mod 13)

42 ≡ 3 ≡ 92

(mod 13)

52 ≡ 12 ≡ 82

(mod 13)

62 ≡ 10 ≡ 72

(mod 13).

Theo đó, 13 có chính xác 6 thặng dư bậc hai là 1, 3, 4, 9, 10 và 12; và nó có 6
thặng dư không bậc hai là 2, 5, 6, 7, 8 và 11.
Ví dụ 2.1.5 cho ta hai nhận định thú vị sau:
• Số nguyên tố 13 có cùng số các thặng dư bậc hai và số thặng dư không bậc

hai (đều bằng 6) và

• chúng lập thành một phân hoạch của tập hợp các thặng dư dương của 13

(xem Hình 2.1)

Hình 2.1: Tập các thặng dư dương của 13


14

Tiếp theo ta sẽ chỉ ra không chỉ riêng số 13 là có cùng con số thặng dư bậc hai
và con số thặng dư không bậc hai. Trước hết ta cần bổ đề sau đây.
Bổ đề 2.1.6. ([5, Lemma 11.1]) Cho p là số nguyên tố lẻ và a là số nguyên sao
cho p a. Khi đó phương trình đồng dư bậc hai
x2 ≡ a (mod p)

(2.4)

hoặc là vô nghiệm hoặc là có đúng hai nghiệm.
Chứng minh. Giả sử α là một nghiệm của phương trình (2.4), ta có α2 ≡ a
(mod p). Đặt β = p − α. Khi đó β 2 = (p − α)2 ≡ (−α)2 ≡ α2 ≡ a (mod p). Do đó
β cũng là một nghiệm của phương trình đã cho. Ngoài ra, ta thấy α và β không

đồng dư với nhau (vì nếu ngược lại β ≡ α (mod p) thì ta có p − α ≡ α (mod p),
tức là −α ≡ α (mod p), nên 2α ≡ 0 (mod p). Nhưng ta biết (2, p) = 1, do đó α ≡ 0
(mod p), điều này mâu thuẫn. Do đó α và p − α là hai đại diện cho hai nghiệm

của phương trình (2.4).
Giả sử phương trình (2.4) còn có nghiệm thứ ba là γ. Khi đó γ 2 ≡ α2 (mod p),
suy ra p | (γ 2 − α2 ). Kéo theo γ ≡ α (mod p) hoặc γ ≡ −α (mod p). Điều đó có
hệ quả là phương trình đồng dư không có nhiều hơn 2 nghiệm.

Định lý 2.1.7. ([5, Theorem 11.1]) Mọi số nguyên tố lẻ p đều có đúng (p − 1)/2
thặng dư bậc hai và (p − 1)/2 thặng dư không bậc hai.
Chứng minh. Giả sử p có k thặng dư bậc hai (không đồng dư nhau từng đôi
một). Theo Bổ đề 2.1.6, mỗi một thặng dư bậc hai a của p đều kéo theo có 2
nghiệm của phương trình đồng dư x2 ≡ a (mod p); nên tổng số các nghiệm của
phương trình đồng dư dạng x2 ≡ a (mod p) là 2k . Nhưng ta đã biết có tất cả p−1
bình phương của các thặng dư dương nhỏ nhất của p (đó là từ 12 , 22 , . . . , (p−1)2 ).
Do đó 2k = p − 1, tức là k = (p − 1)/2. Do vậy, có (p − 1)/2 thặng dư bậc hai và
(p − 1)/2 thặng dư không bậc hai.

Ví dụ 2.1.8. Theo định lý trên ta thấy số p = 17 có (17 − 1)/2 = 8 thặng dư bậc
hai là 1, 4, 9, 16, 8, 2, 15, 13 và có 8 thặng dư không bậc hai là 3, 5, 6, 7, 10, 11, 12, 14,
bởi vì
12 ≡ 162 ≡ 1

(mod 17)

22 ≡ 152 ≡ 4

(mod 17)


15
32 ≡ 142 ≡ 9

(mod 17)

42 ≡ 132 ≡ 16

(mod 17)


52 ≡ 122 ≡ 8

(mod 17)

62 ≡ 112 ≡ 2

(mod 17)

82 ≡ 92 ≡ 13

(mod 17).

72 ≡ 102 ≡ 15

(mod 17)

Tương tự q = 23 có (23 − 1)/2 = 11 thặng dư bậc hai và có 11 thặng dư không
bậc hai.
Ví dụ 2.1.9. Tìm số thặng dư bậc hai của số nguyên tố Fermat.
Giải. Hiện nay, chúng ta mới biết 5 số nguyên tố Fermat ứng với n = 0, 1, 2, 3
và 4. Theo đó, ta có:
0

• Số thặng dư bậc hai của f0 = 22 + 1 = 3 là
3−1
f0 − 1
=
= 1.
2

2
1

• Số thặng dư bậc hai của f1 = 22 + 1 = 5 là
f1 − 1
5−1
=
= 2.
2
2
2

• Số thặng dư bậc hai của f2 = 22 + 1 = 17 là
f2 − 1
17 − 1
=
= 8.
2
2
3

• Số thặng dư bậc hai của f3 = 22 + 1 = 257 là
257 − 1
f3 − 1
=
= 128.
2
2
4


• Số thặng dư bậc hai của f4 = 22 + 1 = 65537 là
4

4

4
f4 − 1
22 + 1 − 1
22
=
=
= 22 −1 = 215 .
2
2
2


16

2.1.3

Tiêu chuẩn Euler và ký hiệu Legendre

Đến giờ ta vẫn chưa có câu trả lời cho câu hỏi “Khi nào thì phương trình
đồng dư x2 ≡ a (mod p) là giải được?” Mục này sẽ tìm hiểu câu trả lời này.
Định lý 2.1.10 (Tiêu chuẩn Euler, [5]). Cho p là số nguyên tố lẻ. Khi đó một
số nguyên dương a (với p

a) là một thặng dư bậc hai của p khi và chỉ khi


a(p−1)/2 ≡ 1 (mod p).

Chứng minh. Giả sử a là một thặng dư bậc hai của p. Khi đó phương trình đồng
dư x2 ≡ a (mod p) có một nghiệm α, tức là α2 ≡ a (mod p). Rõ ràng là (p, α) = 1,
từ đó theo định lý Fermat nhỏ, ta suy ra αp−1 ≡ 1 (mod p). Vì thế cho nên
a(p−1)/2 ≡ (α2 )(p−1)/2 = αp−1 ≡ 1 (mod p).

Ngược lại, giả sử a(p−1)/2 ≡ 1 (mod p). Do đó p có căn nguyên thủy β (tức là
β φ(p) ≡ 1 (mod p) và ordp β = φ(p)). Do đó β (mod p) là phần tử sinh của nhóm

Z∗p , do đó tồn tại k để a ≡ β k (mod p). Kéo theo β k(p−1)/2 ≡ a(p−1)/2 ≡ 1 (mod p).
Vì β là căn nguyên thủy modulo p, nên ordp β = (p − 1) | k(p − 1)/2. Suy ra k
phải là số chẵn, và ta đặt k = 2i. Khi đó a ≡ β 2i ≡ (β i )2 (mod p), nên suy ra a là
thặng dư bậc hai của p.
Ví dụ 2.1.11. Kiểm tra 10 và 7 có là thặng dư bậc hai của 13 hay không?
Giải. Ta có 10(13−1)/2 = 106 ≡ (−3)6 ≡ 1 (mod 13), nên theo tiêu chuẩn Euler ta
suy ra 10 là thặng dư bậc hai của 13. Hệ quả là phương trình x2 ≡ 10 (mod 13)
có nghiệm.
Bây giờ, ta tính toán 7(13−1)/2 (mod 13), ta có 7(13−1)/2 ≡ 76 ≡ (73 )2 ≡ 52 ≡ −1
(mod 13). Vì 76 ≡ 1 (mod 13), nên theo tiêu chuẩn Euler ta suy ra 7 là thặng dư

không bậc hai của 13.
Chú ý 2.1.12. Trong Định lý 2.1.10, giả sử a(p−1)/2 ≡ 1 (mod p). Khi đó a là
thặng dư không bậc hai của p. Khi đó ta có thể nói chính xác những gì về thặng
dư nhỏ nhất của a(p−1)/2 modulo p. Để kết thúc điều này, chú ý rằng theo định lý
Fermat nhỏ ta có ap−1 ≡ 1 (mod p). Vì p lẻ và ap−1 − 1 = [a(p−1)/2 + 1][a(p−1)/2 − 1],
nên ta có hoặc là a(p−1)/2 ≡ 1 (mod p) hoặc là a(p−1)/2 ≡ −1 (mod p). Nhưng vì


17

a(p−1)/2 ≡ 1 (mod p) nên suy ra a(p−1)/2 ≡ −1 (mod p). Do đó, nếu a là thặng dư

không bậc hai của p, thì ta phải có a(p−1)/2 ≡ −1 (mod p).
Ngược lại, cho a là số nguyên thỏa mãn p a và a(p−1)/2 ≡ −1 (mod p). Khi
đó a không thể là thặng dư bậc hai (vì nếu ngược lại, theo tiêu chuẩn Euler,
ta có a(p−1)/2 ≡ 1 (mod p); điều này kéo theo −1 ≡ 1 (mod p), tức là p = 2, mâu
thuẫn). Do đó, nếu a(p−1)/2 ≡ −1 (mod p), thì a phải là thặng dư không bậc hai.
Các lập luận trên đã chứng minh được kết quả sau.
Hệ quả 2.1.13 ([5]). Cho p là số nguyên tố lẻ. Khi đó một số nguyên dương a
(với p a) là thặng dư không bậc hai khi và chỉ khi a(p−1)/2 ≡ −1 (mod p).
Ví dụ 2.1.14. Cho số nguyên tố p = 1213. Ta tính được
5(1213−1)/2 ≡ 5606 ≡ (5101 )6 · 56
≡ (−252)6 · 1069 ≡ 497 · 1069 ≡ −1

(mod 1213).

Theo hệ quả trên, 5 là thặng dư không bậc hai của 1213.
Chú ý 2.1.15. Như vậy ta đã khẳng định được rằng với một số nguyên tố lẻ p
và một số nguyên dương a, ta thấy a là một thặng dư bậc hai của p nếu và chỉ
nếu a(p−1)/2 ≡ 1 (mod p); ta cũng thấy a là thặng dư không bậc hai của p nếu và
chỉ nếu a(p−1)/2 ≡ −1 (mod p).
Theo tiêu chuẩn Euler, phương trình đồng dư (2.4) giải được khi và chỉ khi
a(p−1)/2 ≡ 1 (mod p). Mặc dù Định lý 2.1.10 cho ta một công cụ kiểm tra tính

giải được của phương trình đồng dư, nhưng công cụ này không khả thi khi số
nguyên tố p thực sự lớn. Chẳng hạn, không dễ để áp dụng tiêu chuẩn xác định
tính giải được của phương trình x2 ≡ 3797 (mod 7297). Vì vậy, bây giờ ta cần
thêm công cụ khác để đơn giản hóa nhiệm vụ kiểm tra tính giải được của thặng
dư (2.4), đó là công cụ về “ký hiệu Legendre” sẽ được nghiên cứu tiếp theo sau
đây.

Định nghĩa 2.1.16 ([5]). Cho p là số nguyên tố lẻ và a là số nguyên bất kỳ sao
cho p a. Ký hiệu Legendre (a/p) được xác định bởi công thức
(a/p) =

1
−1

nếu a là thặng dư bậc hai của p
cho trường hợp ngược lại


18

Chú ý rằng, ký hiệu Legendre (a/p) không được xác định nếu p | a. Ví dụ
tiếp theo sẽ minh họa định nghĩa này.
Ví dụ 2.1.17. Trong Ví dụ 2.1.5, ta tìm được 1, 3, 4, 9, 10 và 12 là thặng dư
bậc hai của 13, trong khi 2, 5, 6, 7, 8 và 11 là thặng dư không bậc hai của 13.
Do đó, (1/13) = (3/13) = (4/13) = (9/13) = (10/13) = (12/13) = 1, trong khi đó
(2/13) = (5/13) = (6/13) = (7/13) = (8/13) = (11/13) = −1.

Sử dụng ký hiệu Legendre và kết quả ở Chú ý 2.1.15, ta suy ra rằng (a/p) =
a(p−1)/2 (mod p). Từ Định nghĩa 2.1.16 ta suy ra định lý dưới đây.

Định lý 2.1.18. Cho p là số nguyên tố lẻ. Khi đó số một số nguyên dương a
thỏa mãn p a là một thặng dư bậc hai của p khi và chỉ khi (a/p) = 1.
Chú ý 2.1.19. Nói một cách khác, phương trình x2 ≡ a (mod p) là giải được
khi và chỉ khi (a/p) = 1. Chẳng hạn, theo Ví dụ 2.1.17, ta có (10/13) = 1; nên 10
là một thặng dư bậc hai modulo p. Suy ra phương trình x2 ≡ 10 (mod 13) là giải
được. Nhưng vì (7/13) = −1, nên phương trình x2 ≡ 7 (mod 13) không giải được.
Vì vậy, để tìm xác định tính giải được của phương trình đồng dư x2 ≡ 3797

(mod 7297), ta cần phải tính kí hiệu Legendre (3797/7297). Nhưng bằng cách nào

để tính được nó? Ta không có đủ công cụ để làm việc với kí hiệu này, vì thế ta
biểu diễn ba tính chất cơ bản của kí hiệu Legendre trong định lý dưới đây.
Định lý 2.1.20 ([5]). Cho p là một số nguyên tố lẻ, và a, b là các số nguyên
bất kỳ thỏa mãn p ab. Khi đó
(1) Nếu a ≡ b (mod p) thì (a/p) = (b/p).
(2) (a/p)(b/p) = (ab/p).
(3) (a2 /p) = 1.
Chứng minh. (1) Giả sử a ≡ b (mod p) và phương trình x2 ≡ a (mod p) giải được;
tức là (a/p) = 1. Vì a ≡ b (mod p), nên phương trình x2 ≡ b (mod p) cũng giải
được. Do đó (b/p) = 1 = (a/p). Mặt khác, giả sử x2 ≡ a (mod p) không giải được,
tức là (a/p) = −1. Vì a ≡ b (mod p), nên x2 ≡ b (mod p) cũng không giải được.
Do đó (b/p) = −1 = (a/p). Vì vậy trong cả hai trường hợp ta có (a/p) = (b/p).


19

(2) Theo tiêu chuẩn Euler, ta có (ab/p) ≡ (ab)(p−1)/2 ≡ a(p−1)/2 b(p−1)/2 ≡ (a/p)(b/p)
(mod p). Một lần nữa, vì p lẻ và giá trị của ký hiệu Legendre chỉ là 1 hoặc là −1,

nên (ab/p) ≡ (a/p)(b/p) (mod p) khi và chỉ khi (ab/p) = (a/p)(b/p).
(3) Theo phần (2), ta có (a2 /p) = (a/p)(a/p). Nhưng vì (a/p) = ±1, nên ta có
(a2 /p) = 1 trong cả hai trường hợp. Điều này kết thúc chứng minh.

Chú ý 2.1.21. Tính chất (2) và (3) ở Định lý 2.1.20 giúp ta có thể tính toán
giá trị của ký hiệu Legendre (a2 b/p) với p ab miễn là ta biết giá trị của ký hiệu
Legendre (b/p). Thật vậy, ta có
(a2 b/p) = (a2 /p)(b/p)
= (b/p)


theo tính chất (2) ở Định lý 2.1.20

theo tính chất (3) ở Định lý 2.1.20.

Chẳng hạn, giả sử ta có (7/31) = 1. Khi đó (28/31) = (4/31)(7/31) = 1 · (7/31) =
(7/31) = 1 (ta sẽ thấy cách để tính toán (7/31) ở các phần sau mà không cần

dùng tiêu chuẩn Euler).
Sử dụng tiêu chuẩn Euler, ta có thể nhận diện các số nguyên tố p mà có −1
là một thặng dư bình phương.
Hệ quả 2.1.22 ([5]). Nếu p là số nguyên tố lẻ, thì (−1/p) = (−1)(p−1)/2 . Tức là,
(−1/p) =

1
−1

nếu p ≡ 1 (mod 4)
nếu p ≡ −1 (mod 4).

Chứng minh. Theo tiêu chuẩn Euler, ta có
(−1/p) ≡ (−1)(p−1)/2

(mod p)

= (−1)(p−1)/2 vì (−1)(p−1)/2 = ±1
=

1
−1


=

1
−1

nếu p có dạng 4k + 1
nếu p có dạng 4k + 3
nếu p ≡ 1 (mod 4)
nếu p ≡ −1 (mod 4).

Theo hệ quả trên, ta thấy −1 là thặng dư bậc hai của p khi và chỉ khi p ≡ 1
(mod 4); tức là x2 ≡ p − 1 (mod p) là giải được khi và chỉ khi p ≡ 1 (mod 4).


20

Chẳng hạn, x2 ≡ 12 (mod 13) là giải được, nhưng x2 ≡ 22 (mod 23) không giải
được (vì 23 ≡ 1 (mod 4)).
Hệ quả 2.1.22 có thể được dùng để tính toán các ký hiệu Legendre có dạng
(−a2 /p) như ta chỉ ra ở ví dụ sau.

Ví dụ 2.1.23. Hãy tính (−4/41) và (−9/83)?
Giải. Ta có
(−4/41) = (4/41)(−1/41) theo tính chất (2) của Định lý 2.1.20
= (−1/41) theo tính chất (3) của Định lý 2.1.20
= 1 theo Hệ quả 2.1.22,
(−9/83) = (9/83)(−1/83)
= (−1/83)
= −1 theo Hệ quả 2.1.22.


Tính chất (2) của Định lý 2.1.20 có thể được áp dụng để tính các kí hiệu
Legendre có dạng (q i /p) với p q , như ở hệ quả sau đây.
Hệ quả 2.1.24. Cho p là số nguyên tố lẻ, q là số nguyên tố sao cho p q , và i
là một số nguyên dương. Khi đó (q i /p) = (q/p)i .
Ví dụ sau đây minh họa cho điều này.
Ví dụ 2.1.25. Sử dụng kết quả (5/17) = −1, hãy tính (125/17) và (15625/17).
Giải. Ta có
(125/17) = (53 /17) = (5/17)3 = (−1)3 = −1.
(15625/17) = (56 /17) = (5/17)6 = (−1)6 = 1.

Trở lại Định lý 2.1.20, ta thấy rằng tính chất (2) được mở rộng cho tích hữu
hạn các số nguyên tố không chia hết cho p. Theo đó, ta có kết quả sau đây.


21
n
ei
i=1 pi

Hệ quả 2.1.26. Cho p là số nguyên tố lẻ và
a với (a, p) = 1. Khi đó (a/p) =

là phân tích chính tắc của

n
ei
i=1 (pi /p) .

Chứng minh. Vì (a, p) = 1, (pi , p) = 1 với mọi i. Do đó (pei i /p) = (pi /p)ei theo Hệ

quả 2.1.24. Vì vậy,

n

n

(pei i /p)

(a/p) =
i=1

(pi /p)ei .

=
i=1

Kết quả này được áp dụng để tính (a/p) miễn là ta biết các giá trị (pi /p) với
mọi ước nguyên tố pi của a, như minh họa sau đây.
Ví dụ 2.1.27. Sử dụng kết quả (2/23) = 1 và (5/23) = −1, hay tính (5000/23).
Giải. Ta có 500 = 23 54 . Do đó từ Hệ quả 2.1.26, ta có (500/23) = (2/23)3 (5/23)4 =
13 .(−1)4 = 1.

Chú ý 2.1.28. Làm thế nào để tính được (2/23) = 1 và (5/23) = −1? Ta có thể
sử dụng tiêu chuẩn Euler để tính chúng, nhưng ta muốn tránh điều tẻ nhạt đó.
Thay vào đó, ta có thể tìm hiểu thêm các tính chất của kí hiệu Legendre trong
phần tiếp theo, nhằm giúp ta tính toán (a/p) được thuận lợi hơn. Để làm điều
này, mục tiếp theo ta giới thiệu và chứng minh một tiêu chuẩn của Gauss.

2.1.4


Bổ đề Gauss

Trước khi phát biểu và chứng minh tiêu chuẩn Gauss, ta xét hai ví dụ sau
đây để dần làm rõ về chứng minh tiêu chuẩn này.
Ví dụ 2.1.29 ([5]). Cho p = 23 và a = 5. Ta ký hiệu ν là số các thặng dư dương
nhỏ nhất của 11 = (p − 1)/2 số nguyên 1.5, 2.5, 3.5, . . . , 11.5 modulo p mà lớn hơn
p/2. Hãy tìm ν và xác định xem liệu (5/23) = (−1)ν ?

Giải. Chú ý rằng các thặng dư dương nhỏ nhất trong các số nguyên 1.5, 2.5, 3.5,
4.5, 5.5, 6.5, 7.5, 8.5, 9.5, 10.5, 11.5 modulo 23 lần lượt là 5, 10, 15, 20, 2, 7, 12, 17, 22, 4, 9.

Rõ ràng, có 5 số lớn hơn p/2 = 11, 5, nên ν = 5.
Để tính (5/23), ta áp dụng tiêu chuẩn Euler:
5(p−1)/2 = 511 = (52 )5 · 51 ≡ 25 · 5 ≡ 9 · 5 ≡ −1

(mod 23).


22

Do đó (5/23) = −1. Từ đó ta cũng có (5/23) = (−1)ν .
Ví dụ trên chỉ ra rằng 5 là một thặng dư không bậc hai của 23; hơn nữa,
tính chất bậc hai của 5 modulo 23 được xác định bởi giá trị của ν . Kết quả này
không phải là sự trùng hợp tình cờ mà đó là bản chất như sẽ chỉ ra ở định lý
tiếp theo, trước hết, ta hãy xét thêm một ví dụ nữa.
Ví dụ 2.1.30 ([5]). (Tiếp tục Ví dụ 2.1.29), Ta có ν = 5 thặng dư dương nhỏ
nhất lớn hơn p/2, cụ thể là 12, 15, 17, 20 và 22. Ký hiệu chúng lần lượt là s1 , s2 , s3 và
s5 . Khi đó các số nguyên từ p−s1 tới p−s5 lần lượt là 23−12, 23−15, 23−17, 23−20,

và 23 − 22, cụ thể chúng là 11, 8, 6, 3 và 1; không có hai số nào là đồng dư modulo

23.
Có k = 11 − ν = 11 − 5 = 6 thặng dư, từ r1 đến r6 , là nhỏ hơn p/2, cụ thể là
2, 4, 5, 7, 9 và 10; không có hai số nào là đồng dư modulo 23.

Ngoài ra, không có số nào đồng dư với 11, 8, 6, 3 hoặc 1 modulo 23. Do đó
các thặng dư 2, 4, 5, 7, 9, 10, 11, 8, 6, 3 và 1 là dương và ≤ (p − 1)/2. (Đủ để làm ta
ngạc nhiên, chúng là một hoán vị của các thặng dư từ 1 đến (p − 1)/2 modulo
p).

Bây giờ ta sẵn sàng để đi đến cột mốc tiếp trong tiến trình của ta, cột mốc
này được khám phá bởi Gauss năm 1808. Chứng minh của kết quả này hơi dài
và do đó ta cần kiên nhẫn theo dõi.
Định lý 2.1.31 (Bổ đề Gauss, [5]). Cho p là số nguyên tố lẻ và a là số nguyên sao
cho p a. Cho ν là số các thặng dư dương nhỏ nhất trong các số a, 2a, 3a, . . . (p−1)
2 a
mà lớn hơn p/2. Khi đó (a/p) = (−1)ν .
Chứng minh. Gọi r1 , r2 , . . . , rk là các thặng dư dương nhỏ nhất của các số nguyên
a, 2a, 3a, . . . , [(p − 1)/2]a modulo p mà chúng ≤ p/2, và s1 , s2 , . . . , sν là các thặng

dư trong số đó lớn hơn p/2. Khi đó k + ν = (p − 1)/2.
Xét các số nguyên r1 , r2 , . . . , rk , p − s1 , p − s2 , . . . , p − sν . Mỗi số trong chúng
đều là số dương và nhỏ hơn p/2. Ta cần chỉ ra rằng không có hai số nào trong
chúng đồng dư modulo p.
Đầu tiên, chú ý rằng không có hai số nào trong các số ri đồng dư (vì nếu
ri ≡ rj (mod p) thì ti a ≡ tj a (mod p) với ti , tj sao cho i < j và 1 ≤ ti , tj ≤ (p−1)/2.


×